Biến ngẫu nhiên thống nhất là tổng của hai biến ngẫu nhiên


18

Lấy từ Grimmet và Stirzaker :

Chứng tỏ rằng không thể là trường hợp U = X + YU=X+Y trong đó UU được phân bố đồng đều trên [0,1] và XXYY là độc lập và phân phối giống hệt nhau. Bạn không nên cho rằng X và Y là các biến liên tục.

Một bằng chứng đơn giản bằng cách cũng đủ mâu thuẫn đối với trường hợp XX , YY được giả định rời rạc bởi cho rằng nó luôn luôn có thể tìm thấy một uuu 'u như vậy mà P ( U u + u ' ) P ( U u )P(Uu+u)P(Uu) trong khi P ( X + Y u ) = P ( X + Y u + u )P(X+Yu)=P(X+Yu+u) .

Tuy nhiên, bằng chứng này không mở rộng đến X , YX,Y hoàn toàn liên tục hoặc số ít liên tục. Gợi ý / Nhận xét / Phê bình?


3
Gợi ý : Chức năng đặc trưng là bạn bè của bạn.
Đức hồng y

1
X và Y là iid nên các hàm đặc trưng của chúng phải giống hệt nhau. Mặc dù vậy, bạn cần sử dụng chức năng đặc trưng chứ không phải chức năng tạo thời điểm - mgf không được đảm bảo tồn tại cho X, vì vậy hiển thị mgf có thuộc tính không thể không có nghĩa là không có X. Tất cả RV đều có chức năng đặc trưng, vì vậy nếu bạn cho thấy có một tài sản bất khả thi thì sẽ không có X.
Silverfish

1
Nếu các bản phân phối của XXYY có bất kỳ nguyên tử , nói rằng P { X = một } = P { Y = một } = b > 0P{X=a}=P{Y=a}=b>0 , sau đó P { X + Y = 2 một } b 2 > 0P{X+Y=2a}b2>0 và do đó X + YX+Y không thể được phân phối đồng đều trên [ 0 , 1 ][0,1] . Vì vậy, không cần thiết phải xem xét trường hợp phân phối của XX Y có nguyên tử. Y
Dilip Sarwate

Câu trả lời:


13

Kết quả có thể được chứng minh bằng một hình ảnh: các vùng màu xám có thể nhìn thấy cho thấy phân phối đồng đều không thể bị phân tách thành tổng của hai biến phân phối nhận dạng độc lập.

Ký hiệu

Đặt XY là iid sao cho X + Y có phân phối đồng đều trên [ 0 , 1 ] . Điều này có nghĩa rằng đối với tất cả 0 một b 1 ,XYX+Y[0,1]0ab1

Pr ( a < X + Y b ) = b - a .

Pr(a<X+Yb)=ba.

Sự hỗ trợ thiết yếu của sự phân bố chung của XY do đó là [ 0 , 1 / 2 ] (đối với trường hợp sẽ có khả năng tích cực mà X + Y dối bên ngoài [ 0 , 1 ] ).XY[0,1/2]X+Y[0,1]

Bức tranh

Hãy 0 < ε < 1 / 4 . Xem sơ đồ này cho thấy tổng các biến ngẫu nhiên được tính như thế nào:0<ϵ<1/4

Figure

Phân phối xác suất cơ bản là phân phối chung cho ( X , Y ) . Xác suất của bất kỳ sự kiện nào a < X + Y b được đưa ra bởi tổng xác suất được bao phủ bởi dải đường chéo kéo dài giữa các đường x + y = ax + y = b . Ba ban nhạc như được hiển thị: từ 0 đến ε , xuất hiện như một hình tam giác nhỏ màu xanh ở dưới bên trái; từ 1 / 2 - ε đến 1 / 2(X,Y)a<X+Ybx+y=ax+y=b0ϵ1/2ϵ + ε1/2+ϵ, được hiển thị dưới dạng hình chữ nhật màu xám có hai hình tam giác (màu vàng và màu xanh lá cây); và từ 1 - ε để 1 , xuất hiện như một hình tam giác nhỏ màu đỏ ở phía trên bên phải.1ϵ1

Những gì hình ảnh cho thấy

Bằng cách so sánh tam giác dưới bên trái trong hình với hình vuông bên trái chứa nó và khai thác giả định iid cho XY , rõ ràng làXY

ε = Pr ( X + Y ε ) < Pr ( X ε ) Pr ( Y ε ) = Pr ( X ε ) 2 .

ϵ=Pr(X+Yϵ)<Pr(Xϵ)Pr(Yϵ)=Pr(Xϵ)2.

Lưu ý rằng bất đẳng thức là nghiêm ngặt: không thể bình đẳng vì có một số xác suất dương rằng cả XY đều nhỏ hơn ϵ nhưng tuy nhiênXYϵ X + Y > ϵ .X+Y>ϵ

Tương tự, so sánh tam giác đỏ với hình vuông ở góc trên bên phải,

ε = Pr ( X + Y > 1 - ε ) < Pr ( X > 1 / 2 - ε ) 2 .

ϵ=Pr(X+Y>1ϵ)<Pr(X>1/2ϵ)2.

Cuối cùng, so sánh hai tam giác đối diện ở phía trên bên trái và phía dưới bên phải với dải chéo chứa chúng cho ra một bất đẳng thức nghiêm ngặt khác,

2 ε < 2 Pr ( X ε ) Pr ( X > 1 / 2 - ε ) < Pr ( 1 / 2 - ε < X + Y 1 / 2 + ε ) = 2 ε .

2ϵ<2Pr(Xϵ)Pr(X>1/2ϵ)<Pr(1/2ϵ<X+Y1/2+ϵ)=2ϵ.

Các nảy sinh bất bình đẳng đầu tiên từ trước đó hai (lấy căn bậc hai của họ và nhân họ) trong khi cái thứ hai mô tả (chặt chẽ) bao gồm các hình tam giác trong biên độ và sự bình đẳng trước thể hiện sự thống nhất của X + Y . Kết luận rằng 2 ε < 2 ε là mâu thuẫn minh như XY không thể tồn tại, QED .X+Y2ϵ<2ϵXY


3
(+1) I like this approach. Recovering my back-of-an-envelope from the wastepaper basket I can see I drew the same diagram, except that I didn't mark on the yellow and green triangles inside the band. I did obtain the inequalities for the blue and red triangles. I played around with them and a few other probabilities, but never thought to investigate the probability of the strip, which turns out to be the criticial step. I wonder what thought process might have motivated this insight?
Silverfish

Trên thực tế, nơi @whuber có các hình tam giác màu vàng và màu xanh lá cây, tôi đã vẽ trên các hình vuông (tôi đã phân hủy hiệu quả [ 0 , 0,5 ] 2 thành một lưới). Nhìn vào các bước mà "mô tả (chặt chẽ) bao gồm các hình tam giác trong ban nhạc", 2 Pr ( X ε ) Pr ( X > 1 / 2 - ε ) < Pr ( 1 / 2 - ε < X + Y 1 / 2 +[0,0.5]2 ε )2Pr(Xϵ)Pr(X>1/2ϵ)<Pr(1/2ϵ<X+Y1/2+ϵ), I wonder whether this would actually be geometrically more natural with squares capping the band than triangles?
Silverfish

1
@Silver I was reminded of an analysis of sums of uniform distributions I posted a couple of years ago. That suggested visualizing the sum X+YX+Y geometrically. It was immediately evident that a lot of probability had to be concentrated near the corners (0,0)(0,0) and (1/2,1/2)(1/2,1/2) in order for the sum to be uniform and for relatively little probability to be near the center diagonal X+Y=1/2X+Y=1/2. That led to the diagram, which I redrew in Mathematica. At that point the answer wrote itself. Yes, using squares in the center band might be neater.
whuber

Thanks! "Note that the inequality is strict: equality is not possible because there is some positive probability that either of XX or YY is less than ϵϵ but nevertheless X+Y>ϵX+Y>ϵ." I'm not sure I follow this. It seems to me the aim here is to show Pr(X+Yϵ)<Pr(XϵYϵ)Pr(X+Yϵ)<Pr(XϵYϵ), doesn't this require a positive probability for some event AA in which both of XX and YY are less than or equal to ϵϵ and yet X+Y>ϵX+Y>ϵ? It is the "either of" vs "both of" I'm vacillating over.
Silverfish

@Silverfish Thank you; I did not express that as I had intended. You are correct: the language is intended essentially to describe the portion of a little square not inside the triangle.
whuber

10

I tried finding a proof without considering characteristic functions. Excess kurtosis does the trick. Here's the two-line answer: Kurt(U)=Kurt(X+Y)=Kurt(X)/2Kurt(U)=Kurt(X+Y)=Kurt(X)/2 since XX and YY are iid. Then Kurt(U)=1.2Kurt(U)=1.2 implies Kurt(X)=2.4Kurt(X)=2.4 which is a contradiction as Kurt(X)2Kurt(X)2 for any random variable.

Rather more interesting is the line of reasoning that got me to that point. XX (and YY) must be bounded between 0 and 0.5 - that much is obvious, but helpfully means that its moments and central moments exist. Let's start by considering the mean and variance: E(U)=0.5E(U)=0.5 and Var(U)=112Var(U)=112. If XX and YY are identically distributed then we have:

E(X+Y)=E(X)+E(Y)=2E(X)=0.5

E(X+Y)=E(X)+E(Y)=2E(X)=0.5

So E(X)=0.25E(X)=0.25. For the variance we additionally need to use independence to apply:

Var(X+Y)=Var(X)+Var(Y)=2Var(X)=112

Var(X+Y)=Var(X)+Var(Y)=2Var(X)=112

Hence Var(X)=124Var(X)=124 and σX=1260.204σX=1260.204. Wow! That is a lot of variation for a random variable whose support ranges from 0 to 0.5. But we should have expected that, since the standard deviation isn't going to scale in the same way that the mean did.

Now, what's the largest standard deviation that a random variable can have if the smallest value it can take is 0, the largest value it can take is 0.5, and the mean is 0.25? Collecting all the probability at two point masses on the extremes, 0.25 away from the mean, would clearly give a standard deviation of 0.25. So our σXσX is large but not impossible. (I hoped to show that this implied too much probability lay in the tails for X+YX+Y to be uniform, but I couldn't get anywhere with that on the back of an envelope.)

Second moment considerations almost put an impossible constraint on XX so let's consider higher moments. What about Pearson's moment coefficient of skewness, γ1=E(XμX)3σ3X=κ3κ3/22γ1=E(XμX)3σ3X=κ3κ3/22? This exists since the central moments exist and σX0σX0. It is helpful to know some properties of the cumulants, in particular applying independence and then identical distribution gives:

κi(U)=κi(X+Y)=κi(X)+κi(Y)=2κi(X)

κi(U)=κi(X+Y)=κi(X)+κi(Y)=2κi(X)

This additivity property is precisely the generalisation of how we dealt with the mean and variance above - indeed, the first and second cumulants are just κ1=μκ1=μ and κ2=σ2κ2=σ2.

Then κ3(U)=2κ3(X)κ3(U)=2κ3(X) and (κ2(U))3/2=(2κ2(X))3/2=23/2(κ2(X))3/2(κ2(U))3/2=(2κ2(X))3/2=23/2(κ2(X))3/2. The fraction for γ1γ1 cancels to yield Skew(U)=Skew(X+Y)=Skew(X)/2Skew(U)=Skew(X+Y)=Skew(X)/2. Since the uniform distribution has zero skewness, so does XX, but I can't see how a contradiction arises from this restriction.

So instead, let's try the excess kurtosis, γ2=κ4κ22=E(XμX)4σ4X3γ2=κ4κ22=E(XμX)4σ4X3. By a similar argument (this question is self-study, so try it!), we can show this exists and obeys:

Kurt(U)=Kurt(X+Y)=Kurt(X)/2

Kurt(U)=Kurt(X+Y)=Kurt(X)/2

The uniform distribution has excess kurtosis 1.21.2 so we require XX to have excess kurtosis 2.42.4. But the smallest possible excess kurtosis is 22, which is achieved by the Binomial(1,12)Binomial(1,12) Bernoulli distribution.


2
(+1) This is a quite clever approach, which was new to me. Thanks. Note that some of your analysis could have been streamlined by considering a uniform centered at zero. (The equivalence of the problem is immediate.) That would have immediately told you that considering skew was a dead-end.
cardinal

@cardinal: I knew the skew was a dead-end before I worked on it. The purpose was expository: it's a self-study question so I didn't want to solve it in full! Rather I wanted to leave a hint on how to deal with the next level up...
Silverfish

@cardinal: I was in two minds whether to center or not. I did back-of-envelope calculations more conveniently, but in the final analysis we just need (1) a simple case of the general result that Kurt(X1+...+Xn)=1nKurt(X)Kurt(X1+...+Xn)=1nKurt(X) for iid XiXi, (2) that Kurt(U)=1.2Kurt(U)=1.2 for any uniform distribution, and (3) Kurt(X)Kurt(X) exists since XX is bounded and σX0σX0 (which is trivial, else σU=0σU=0). So none of the key results actually required centering, though bits may have looked less ugly!
Silverfish

Yes, the word "streamlined" was carefully chosen. :-) I did not intend my comment to be read as criticism of your exposition. Cheers.
cardinal

@cardinal Incidentally, variance considerations alone almost worked, but the uniform isn't quite spread out enough. With a bit more probability mass nearer the extremes, e.g. fT(t)=12t2fT(t)=12t2 on [-0.5, 0.5], then Var(T)=.15Var(T)=.15 and if T=X1+X2T=X1+X2 then σX=.15/20.27>0.25σX=.15/20.27>0.25 which is impossible as XX is bounded by -0.25 and 0.25. Of course, you will see immediately how this relates to the present example! I wonder if the approach generalises, I'm sure other bounded RVs can't be decomposed into sums but require even higher moments investigated to find the contradiction.
Silverfish
Khi sử dụng trang web của chúng tôi, bạn xác nhận rằng bạn đã đọc và hiểu Chính sách cookieChính sách bảo mật của chúng tôi.
Licensed under cc by-sa 3.0 with attribution required.